digital camera

This topic has expert replies
Master | Next Rank: 500 Posts
Posts: 186
Joined: Fri May 28, 2010 1:05 am
Thanked: 11 times

digital camera

by jube » Sat Jun 19, 2010 5:44 am
The number of digital cameras sold in the past year increased dramatically over the number of digital cameras sold in the previous year. The Chief Executive Officer of the corporation that produces Digicam, a popular brand of digital camera, proposes to increase production of Digicam products.

Which of the following statements, if true, would most strengthen the argument that the Chief Executive Officer's plan will NOT work?

A. As a percentage of digital cameras sold, Digicam ' s market share decreased over the past twelve months.
B. The lag time between production and shipment of Digicams has decreased during the last twelve months.
C. Celebrity endorsements have increased Digicam ' s brand recognition, but not awareness of the parent corporation.
D. Digicam is one of the brands most responsible for the increase in digital camera sales during the last twelve months.
E. Sales of Digicam decreased during the last twelve months, despite widespread distribution of coupons in stores and newspapers and on the Internet.

User avatar
Legendary Member
Posts: 535
Joined: Fri Jun 08, 2007 2:12 am
Thanked: 87 times
Followed by:5 members
GMAT Score:730

by hardik.jadeja » Sat Jun 19, 2010 5:53 am
Its a tight call between A and E. But my choice is E.

Master | Next Rank: 500 Posts
Posts: 186
Joined: Fri May 28, 2010 1:05 am
Thanked: 11 times

by jube » Sat Jun 19, 2010 5:56 am
hardik.jadeja wrote:Its a tight call between A and E. But my choice is E.
OA: E

So why is A wrong? (I chose A :) )

User avatar
Legendary Member
Posts: 535
Joined: Fri Jun 08, 2007 2:12 am
Thanked: 87 times
Followed by:5 members
GMAT Score:730

by hardik.jadeja » Sat Jun 19, 2010 6:03 am
jube wrote: OA: E

So why is A wrong? (I chose A :) )
Option E correctly mentions that sales of Digicam decreased during the last twelve months, so to increase the production of Digicam products is going to result in losses.

A is wrong because even though Digicam's market share decreased over the past twelve months, it is possible that their own sales have increased over the previous year.

think this way,

Lets say last year 100 digital cameras were sold in the country. And digicam had sold 10 out of 100, so digicam's market share was 10%.

Now lets say this year 200 digital cameras have been sold and digicam has sold 15. Notice that digicam's market share has declined here (from 10% to 7.5%) but their own sales have increased over the previous year(50% increase). So to increase the production of Digicam products further still makes sense.

Hope that helps..

User avatar
Legendary Member
Posts: 758
Joined: Sat Aug 29, 2009 9:32 pm
Location: Bangalore,India
Thanked: 67 times
Followed by:2 members

by sumanr84 » Sat Jun 19, 2010 6:17 am
@hardik - A very plausible explanation.,I must say.
I am on a break !!

Master | Next Rank: 500 Posts
Posts: 135
Joined: Sat Jun 20, 2009 10:59 am
Thanked: 4 times

by missionGMAT007 » Sat Jun 19, 2010 6:33 am
IMO E

Legendary Member
Posts: 2330
Joined: Fri Jan 15, 2010 5:14 am
Thanked: 56 times
Followed by:26 members

by mundasingh123 » Fri Dec 31, 2010 3:43 am
jube wrote:The number of digital cameras sold in the past year increased dramatically over the number of digital cameras sold in the previous year. The Chief Executive Officer of the corporation that produces Digicam, a popular brand of digital camera, proposes to increase production of Digicam products.

Which of the following statements, if true, would most strengthen the argument that the Chief Executive Officer's plan will NOT work?

A. As a percentage of digital cameras sold, Digicam ' s market share decreased over the past twelve months.
B. The lag time between production and shipment of Digicams has decreased during the last twelve months.
C. Celebrity endorsements have increased Digicam ' s brand recognition, but not awareness of the parent corporation.
D. Digicam is one of the brands most responsible for the increase in digital camera sales during the last twelve months.
E. Sales of Digicam decreased during the last twelve months, despite widespread distribution of coupons in stores and newspapers and on the Internet.
Whats the source?

User avatar
Legendary Member
Posts: 752
Joined: Sun Sep 12, 2010 2:47 am
Thanked: 20 times
Followed by:10 members
GMAT Score:700

by prachich1987 » Fri Dec 31, 2010 4:01 am
I would have marked E

jube what's the OA?

Junior | Next Rank: 30 Posts
Posts: 19
Joined: Mon Dec 20, 2010 8:56 pm
Thanked: 1 times

by captcha » Fri Dec 31, 2010 5:45 am
IMO E

Between A & E

A ---> market share decreased, but sales can increase.

E ---> sales decreased. therefore, CEO's plan will fail

User avatar
Senior | Next Rank: 100 Posts
Posts: 48
Joined: Mon Oct 12, 2009 5:15 am
Thanked: 5 times
Followed by:1 members

by lavinia » Sun Jan 02, 2011 3:29 pm
Hi Jube,

As you can see, GMAT wants to deceive you with this answer choice. Because you are looking for something that decreased, A is an appealing answer. Always be careful when you have numbers in the Stimulus and percentage in the Answer choice. Most of the time are wrong.
jube wrote:
hardik.jadeja wrote:Its a tight call between A and E. But my choice is E.
OA: E

So why is A wrong? (I chose A :) )

User avatar
Legendary Member
Posts: 1255
Joined: Fri Nov 07, 2008 2:08 pm
Location: St. Louis
Thanked: 312 times
Followed by:90 members

by Tani » Sun Jan 02, 2011 9:37 pm
The real problem with this question is that we aren't told what "work" is. We are told he wants to make more cameras, but not what that is supposed to accomplish. We are never told that they don't have enough to sell.

If we assume he wants more revenue then his "plan" must contribute to that. Making more cameras will only increase revenue of they can sell more. E tells us they probably can't.

The problem with A is numbers versus percent. Assume in year 1 there were 100 cameras sold and Digicam sold 50. That's a 50% share.

Assume in the next year there are 200 cameras sold and Digicam sells 75. Digicam is up 50%, but it's share is down to 37.5%.
Tani Wolff

User avatar
Master | Next Rank: 500 Posts
Posts: 163
Joined: Tue Nov 24, 2009 7:56 am
Location: Philadelphia
Thanked: 13 times
Followed by:4 members
GMAT Score:660

by chendawg » Thu Jan 20, 2011 10:51 am
What's the source?

I don't think this is very a GMAT like question, because it doesn't really have a conclusion. We're just told that cameras sold last yr was more than the yr before that. Then we're told the CEO of the corporation of Digicam, a popular camera, proposes to increase the # of PRODUCTS (not cameras) made by the corporation. There's no conclusion stated, so how can we argue if the plan will "work", when he didn't really state the purpose? Is it to increase profits? Or is he TRYING to lose money? Maybe he shorted the company stock and he's trying to lower the stock price for personal gains. We don't know, we have to assume the purpose of the plan.

Legendary Member
Posts: 608
Joined: Sun Jun 19, 2011 11:16 am
Thanked: 37 times
Followed by:8 members

by saketk » Thu Oct 13, 2011 11:14 am
i too got down to A and E but A can be eliminated because of this part 'As a percentage of digital cameras sold' -- someone here explained it already so I am not going to do that again :)

IMO E

Master | Next Rank: 500 Posts
Posts: 101
Joined: Thu Aug 25, 2011 9:39 pm

by prashant misra » Sat Oct 29, 2011 10:10 am
i was also confused between A and C and ended up choosing the wrong answer option A but now i have got the point why A is wrong

Master | Next Rank: 500 Posts
Posts: 150
Joined: Thu May 05, 2011 10:04 am
Thanked: 5 times
Followed by:4 members

by thestartupguy » Sat Oct 29, 2011 12:57 pm
Clearly the answer is E